Cảm giác như thế nào khi quay quanh một lỗ đen hợp nhất?


10

Phát hiện thứ 4 gần đây về sóng hấp dẫn trong đó khối lượng lỗ đen gấp 31 và 25 lần khối lượng mặt trời và chúng giải phóng 3 khối năng lượng mặt trời dưới dạng sóng hấp dẫn, khiến tôi tự hỏi cảm giác như thế nào khi quay quanh lỗ đen nhị phân này hệ thống trên một hành tinh tại 1AU khi vụ sáp nhập xảy ra?


3
Bài viết này giải quyết câu hỏi của bạn, được viết bởi một nhà vật lý. forbes.com/sites/briankoberlein/2016/02/13/... Và đây là một câu hỏi rất giống nhau: physics.stackexchange.com/questions/235285/...
userLTK

1
câu trả lời tại vật lý rất khác với câu trả lời của @ t.sand ...
J. Chomel

2
Các câu trả lời trong stackexchange vật lý là hoàn toàn không thỏa đáng. Những người liên quan đến năng lượng thậm chí không đề cập đến việc lực hấp dẫn tương tác yếu đến mức bản thân 3 khối năng lượng hấp dẫn của mặt trời sẽ làm rất ít cho con người vì kích thước của con người và kích thước của hằng số hấp dẫn. Câu trả lời được chấp nhận là khá vô ích khi các biến dạng không thời gian không giống như nén và kéo dài thông thường và không có bằng chứng nào cho thấy sóng hấp dẫn trơn tru làm biến dạng vật chất có thể gây hại cho con người ở kích thước 0,01% của con người.
ACAC

Tôi ngạc nhiên khi không ai nhận ra quan điểm của @ ACAC rằng các GW là gợn sóng trong không thời gian , và sự biến dạng này về cơ bản là khác nhau đối với việc nén hoặc kéo dài các vật thể trong không gian thông thường. Một điều khác bị bỏ qua là GW di chuyển với tốc độ ánh sáng: không thể "cảm nhận" được một gợn sóng ở tốc độ đó.
Chappo đã không quên Monica

Câu trả lời:


3

Nó có thể gây thiệt hại nghiêm trọng nếu mọi thứ vừa phải.

Ba khối năng lượng mặt trời là khoảng 5x10 47 Joules. 1 AU khoảng 1,5x10 13 cm. Do đó, diện tích của hình cầu là khoảng 4x10 27 cm 2 . Đó là một dòng 10 20 joules cm -2 . Đó là rất nhiều năng lượng để gây sát thương ... nếu nó có thể kết đôi với bạn.

Biến dạng của GW đo được tại Trái đất là khoảng 10 -22 . Strain giảm xuống khi khoảng cách nghịch đảo và tỷ lệ của gigaparsec so với AU là khoảng 10 14 , do đó, biến dạng tại 1AU từ sáp nhập sẽ vào khoảng 10 -8 . Chính nó, điều này có thể sẽ không làm hại bạn.

Nhưng đó không phải là một biến dạng đơn giản, nó dao động với tần số quỹ đạo của các lỗ đen hợp nhất và bất cứ thứ gì có cộng hưởng vật lý ở tần số đó đều có thể lấy năng lượng, có thể là năng lượng đáng kể từ các trường GW.

(Lưu ý rằng điều này phụ thuộc vào định hướng. Sự cộng hưởng cơ học cần phải phù hợp với GW và các GW không được phát ra theo mọi hướng, vì vậy bạn muốn (hoặc, có lẽ, không muốn!) Nằm trong mặt phẳng của sáp nhập. )

Lập luận rằng các GW không thể ký gửi năng lượng là hoàn toàn sai - nếu đó là sự thật, LIGO không thể phát hiện ra chúng! - nhưng nếu bạn tin vào điều đó, bạn đã ở trong một công ty tuyệt vời: Phải mất hàng thập kỷ trước khi anh ta tin chắc rằng các GW thực sự mang năng lượng!


1
108

2

Nó là khá đẹp để tưởng tượng một điều kiện như vậy.

Điều đầu tiên là chúng ta không thể quan sát chúng bằng mắt, vì sẽ không có sự phản chiếu ánh sáng từ các lỗ đen.

Khi người quan sát ở trạng thái nghỉ ngơi liên quan đến việc hợp nhất các lỗ đen: bây giờ khi các lỗ đen hợp nhất, sóng hấp dẫn của không gian thời gian xung quanh sẽ bắt đầu tăng lên và biên độ của chúng cũng sẽ tăng lên. Nó ảnh hưởng đến người quan sát, vì sóng bắt đầu đẩy người quan sát ra khỏi các lỗ đen. Nhưng nó không ảnh hưởng đến hình ảnh quan sát, vì ánh sáng cũng sẽ bị bẻ cong trong sóng hấp dẫn. Ngoài ra sự giãn nở thời gian sẽ xảy ra do sóng hấp dẫn, nhưng những gì thực sự xảy ra khi chúng hợp nhất, tôi không thể tưởng tượng được.


Tôi nghĩ bạn đã đúng, rằng sóng hấp dẫn mạnh như vậy có tác dụng đẩy, tuy nhiên làm thế nào bạn có thể tưởng tượng điều này mà không có kiến ​​thức sâu về Thuyết tương đối rộng? Đó là một câu hỏi khó. :-)
peterh - Phục hồi Monica

1
10401050

cảm ơn bạn đã trả lời tôi sẽ cố gắng hết sức để hoàn thành câu trả lời. và tôi hy vọng lần sau tôi sẽ không làm bạn thất vọng. thực ra tôi đang học về thuyết tương đối từ tiêu chuẩn thứ 9 từ ống của bạn (kênh Space-Time).

1
Đồng ý. Bạn không cần thuyết tương đối cho điều đó, chỉ cần xem xét bảo toàn xung động những gì đúng trong Thuyết tương đối rộng.
peterh - Phục hồi Monica
Khi sử dụng trang web của chúng tôi, bạn xác nhận rằng bạn đã đọc và hiểu Chính sách cookieChính sách bảo mật của chúng tôi.
Licensed under cc by-sa 3.0 with attribution required.